A power series with decreasing positive coefficients has no zeroes in the disk

The name of the pictureThe name of the pictureThe name of the pictureClash Royale CLAN TAG#URR8PPP












8












$begingroup$


Let $sum_n=0^infty a_n z^n$ be a formal complex power series, with $a_n$ strictly decreasing to 1 as $nto infty$. It is easy to see that the radius of convergence is 1, so that this power series represents a function $f$ holomorphic on the disk. The question is to



Show that f has no zeroes on the disk.



I've been looking at this a while and feel and tried a few different things.



$cdot$ One easy observation is the similarity of this series with $frac11-z$, whose coefficients do not strictly decrease but which otherwise is an example of the thing to be proven.



$cdot$ Rouché's theorem doesn't seem to have an immediate application—I've tried comparing $f$ to $frac11-z$ or to its partial sums by Rouché.



$cdot$ If the partial sums had no zeroes, then neither would $f$ by Hurwitz's theorem



$cdot$ If we could show the function had positive real part we'd obviously be done, which seems highly plausible—the condition gives that $f(-1) = sum a_n cospi n > 0$, but I don't know how to extend this argument to arguments other than $pi$. Besides, if this technique were to work, it seems it would rely most on algebraic manipulations of power series, and I would vastly prefer a classical complex analytic approach (argument principle, Rouché's theorem, etc)



Please advise!










share|cite|improve this question









$endgroup$







  • 1




    $begingroup$
    Possible duplicate of Let $(a_n)_n geq 0$ be a strictly decreasing sequence of positive real numbers , and let $z in mathbb C$ , $|z| < 1$.
    $endgroup$
    – Martin R
    Feb 19 at 8:02
















8












$begingroup$


Let $sum_n=0^infty a_n z^n$ be a formal complex power series, with $a_n$ strictly decreasing to 1 as $nto infty$. It is easy to see that the radius of convergence is 1, so that this power series represents a function $f$ holomorphic on the disk. The question is to



Show that f has no zeroes on the disk.



I've been looking at this a while and feel and tried a few different things.



$cdot$ One easy observation is the similarity of this series with $frac11-z$, whose coefficients do not strictly decrease but which otherwise is an example of the thing to be proven.



$cdot$ Rouché's theorem doesn't seem to have an immediate application—I've tried comparing $f$ to $frac11-z$ or to its partial sums by Rouché.



$cdot$ If the partial sums had no zeroes, then neither would $f$ by Hurwitz's theorem



$cdot$ If we could show the function had positive real part we'd obviously be done, which seems highly plausible—the condition gives that $f(-1) = sum a_n cospi n > 0$, but I don't know how to extend this argument to arguments other than $pi$. Besides, if this technique were to work, it seems it would rely most on algebraic manipulations of power series, and I would vastly prefer a classical complex analytic approach (argument principle, Rouché's theorem, etc)



Please advise!










share|cite|improve this question









$endgroup$







  • 1




    $begingroup$
    Possible duplicate of Let $(a_n)_n geq 0$ be a strictly decreasing sequence of positive real numbers , and let $z in mathbb C$ , $|z| < 1$.
    $endgroup$
    – Martin R
    Feb 19 at 8:02














8












8








8


2



$begingroup$


Let $sum_n=0^infty a_n z^n$ be a formal complex power series, with $a_n$ strictly decreasing to 1 as $nto infty$. It is easy to see that the radius of convergence is 1, so that this power series represents a function $f$ holomorphic on the disk. The question is to



Show that f has no zeroes on the disk.



I've been looking at this a while and feel and tried a few different things.



$cdot$ One easy observation is the similarity of this series with $frac11-z$, whose coefficients do not strictly decrease but which otherwise is an example of the thing to be proven.



$cdot$ Rouché's theorem doesn't seem to have an immediate application—I've tried comparing $f$ to $frac11-z$ or to its partial sums by Rouché.



$cdot$ If the partial sums had no zeroes, then neither would $f$ by Hurwitz's theorem



$cdot$ If we could show the function had positive real part we'd obviously be done, which seems highly plausible—the condition gives that $f(-1) = sum a_n cospi n > 0$, but I don't know how to extend this argument to arguments other than $pi$. Besides, if this technique were to work, it seems it would rely most on algebraic manipulations of power series, and I would vastly prefer a classical complex analytic approach (argument principle, Rouché's theorem, etc)



Please advise!










share|cite|improve this question









$endgroup$




Let $sum_n=0^infty a_n z^n$ be a formal complex power series, with $a_n$ strictly decreasing to 1 as $nto infty$. It is easy to see that the radius of convergence is 1, so that this power series represents a function $f$ holomorphic on the disk. The question is to



Show that f has no zeroes on the disk.



I've been looking at this a while and feel and tried a few different things.



$cdot$ One easy observation is the similarity of this series with $frac11-z$, whose coefficients do not strictly decrease but which otherwise is an example of the thing to be proven.



$cdot$ Rouché's theorem doesn't seem to have an immediate application—I've tried comparing $f$ to $frac11-z$ or to its partial sums by Rouché.



$cdot$ If the partial sums had no zeroes, then neither would $f$ by Hurwitz's theorem



$cdot$ If we could show the function had positive real part we'd obviously be done, which seems highly plausible—the condition gives that $f(-1) = sum a_n cospi n > 0$, but I don't know how to extend this argument to arguments other than $pi$. Besides, if this technique were to work, it seems it would rely most on algebraic manipulations of power series, and I would vastly prefer a classical complex analytic approach (argument principle, Rouché's theorem, etc)



Please advise!







complex-analysis power-series






share|cite|improve this question













share|cite|improve this question











share|cite|improve this question




share|cite|improve this question










asked Feb 18 at 16:04









Van LatimerVan Latimer

363110




363110







  • 1




    $begingroup$
    Possible duplicate of Let $(a_n)_n geq 0$ be a strictly decreasing sequence of positive real numbers , and let $z in mathbb C$ , $|z| < 1$.
    $endgroup$
    – Martin R
    Feb 19 at 8:02













  • 1




    $begingroup$
    Possible duplicate of Let $(a_n)_n geq 0$ be a strictly decreasing sequence of positive real numbers , and let $z in mathbb C$ , $|z| < 1$.
    $endgroup$
    – Martin R
    Feb 19 at 8:02








1




1




$begingroup$
Possible duplicate of Let $(a_n)_n geq 0$ be a strictly decreasing sequence of positive real numbers , and let $z in mathbb C$ , $|z| < 1$.
$endgroup$
– Martin R
Feb 19 at 8:02





$begingroup$
Possible duplicate of Let $(a_n)_n geq 0$ be a strictly decreasing sequence of positive real numbers , and let $z in mathbb C$ , $|z| < 1$.
$endgroup$
– Martin R
Feb 19 at 8:02











1 Answer
1






active

oldest

votes


















10












$begingroup$

Let us consider $f(z) =(1-z)sumlimits_n=0^infty a_n z^n =a_0+sumlimits_n=1^infty (a_n-a_n-1)z^n$. Suppose $f(re^itheta)=0$ for $rle 1$. Then it follows
$$
a_0 =sum_n=1^infty (a_n-1-a_n)r^ne^intheta,
$$
hence
$$
a_0=left|sum_n=1^infty (a_n-1-a_n)r^ne^inthetaright|le sum_n=1^infty (a_n-1-a_n)r^nlesum_n=1^infty(a_n-1-a_n)=a_0-1,
$$
which leads to a contradiction. Since $f$ has no zeros on the unit disk, neither does $fracf(z)1-z=sumlimits_n=0^infty a_n z^n$.






share|cite|improve this answer











$endgroup$












  • $begingroup$
    That's beautiful, thank you. I looked at the series (1-z) sum a_n z^n but didn't see how to get this contradiction.
    $endgroup$
    – Van Latimer
    Feb 18 at 16:50










Your Answer





StackExchange.ifUsing("editor", function ()
return StackExchange.using("mathjaxEditing", function ()
StackExchange.MarkdownEditor.creationCallbacks.add(function (editor, postfix)
StackExchange.mathjaxEditing.prepareWmdForMathJax(editor, postfix, [["$", "$"], ["\\(","\\)"]]);
);
);
, "mathjax-editing");

StackExchange.ready(function()
var channelOptions =
tags: "".split(" "),
id: "69"
;
initTagRenderer("".split(" "), "".split(" "), channelOptions);

StackExchange.using("externalEditor", function()
// Have to fire editor after snippets, if snippets enabled
if (StackExchange.settings.snippets.snippetsEnabled)
StackExchange.using("snippets", function()
createEditor();
);

else
createEditor();

);

function createEditor()
StackExchange.prepareEditor(
heartbeatType: 'answer',
autoActivateHeartbeat: false,
convertImagesToLinks: true,
noModals: true,
showLowRepImageUploadWarning: true,
reputationToPostImages: 10,
bindNavPrevention: true,
postfix: "",
imageUploader:
brandingHtml: "Powered by u003ca class="icon-imgur-white" href="https://imgur.com/"u003eu003c/au003e",
contentPolicyHtml: "User contributions licensed under u003ca href="https://creativecommons.org/licenses/by-sa/3.0/"u003ecc by-sa 3.0 with attribution requiredu003c/au003e u003ca href="https://stackoverflow.com/legal/content-policy"u003e(content policy)u003c/au003e",
allowUrls: true
,
noCode: true, onDemand: true,
discardSelector: ".discard-answer"
,immediatelyShowMarkdownHelp:true
);



);













draft saved

draft discarded


















StackExchange.ready(
function ()
StackExchange.openid.initPostLogin('.new-post-login', 'https%3a%2f%2fmath.stackexchange.com%2fquestions%2f3117764%2fa-power-series-with-decreasing-positive-coefficients-has-no-zeroes-in-the-disk%23new-answer', 'question_page');

);

Post as a guest















Required, but never shown

























1 Answer
1






active

oldest

votes








1 Answer
1






active

oldest

votes









active

oldest

votes






active

oldest

votes









10












$begingroup$

Let us consider $f(z) =(1-z)sumlimits_n=0^infty a_n z^n =a_0+sumlimits_n=1^infty (a_n-a_n-1)z^n$. Suppose $f(re^itheta)=0$ for $rle 1$. Then it follows
$$
a_0 =sum_n=1^infty (a_n-1-a_n)r^ne^intheta,
$$
hence
$$
a_0=left|sum_n=1^infty (a_n-1-a_n)r^ne^inthetaright|le sum_n=1^infty (a_n-1-a_n)r^nlesum_n=1^infty(a_n-1-a_n)=a_0-1,
$$
which leads to a contradiction. Since $f$ has no zeros on the unit disk, neither does $fracf(z)1-z=sumlimits_n=0^infty a_n z^n$.






share|cite|improve this answer











$endgroup$












  • $begingroup$
    That's beautiful, thank you. I looked at the series (1-z) sum a_n z^n but didn't see how to get this contradiction.
    $endgroup$
    – Van Latimer
    Feb 18 at 16:50















10












$begingroup$

Let us consider $f(z) =(1-z)sumlimits_n=0^infty a_n z^n =a_0+sumlimits_n=1^infty (a_n-a_n-1)z^n$. Suppose $f(re^itheta)=0$ for $rle 1$. Then it follows
$$
a_0 =sum_n=1^infty (a_n-1-a_n)r^ne^intheta,
$$
hence
$$
a_0=left|sum_n=1^infty (a_n-1-a_n)r^ne^inthetaright|le sum_n=1^infty (a_n-1-a_n)r^nlesum_n=1^infty(a_n-1-a_n)=a_0-1,
$$
which leads to a contradiction. Since $f$ has no zeros on the unit disk, neither does $fracf(z)1-z=sumlimits_n=0^infty a_n z^n$.






share|cite|improve this answer











$endgroup$












  • $begingroup$
    That's beautiful, thank you. I looked at the series (1-z) sum a_n z^n but didn't see how to get this contradiction.
    $endgroup$
    – Van Latimer
    Feb 18 at 16:50













10












10








10





$begingroup$

Let us consider $f(z) =(1-z)sumlimits_n=0^infty a_n z^n =a_0+sumlimits_n=1^infty (a_n-a_n-1)z^n$. Suppose $f(re^itheta)=0$ for $rle 1$. Then it follows
$$
a_0 =sum_n=1^infty (a_n-1-a_n)r^ne^intheta,
$$
hence
$$
a_0=left|sum_n=1^infty (a_n-1-a_n)r^ne^inthetaright|le sum_n=1^infty (a_n-1-a_n)r^nlesum_n=1^infty(a_n-1-a_n)=a_0-1,
$$
which leads to a contradiction. Since $f$ has no zeros on the unit disk, neither does $fracf(z)1-z=sumlimits_n=0^infty a_n z^n$.






share|cite|improve this answer











$endgroup$



Let us consider $f(z) =(1-z)sumlimits_n=0^infty a_n z^n =a_0+sumlimits_n=1^infty (a_n-a_n-1)z^n$. Suppose $f(re^itheta)=0$ for $rle 1$. Then it follows
$$
a_0 =sum_n=1^infty (a_n-1-a_n)r^ne^intheta,
$$
hence
$$
a_0=left|sum_n=1^infty (a_n-1-a_n)r^ne^inthetaright|le sum_n=1^infty (a_n-1-a_n)r^nlesum_n=1^infty(a_n-1-a_n)=a_0-1,
$$
which leads to a contradiction. Since $f$ has no zeros on the unit disk, neither does $fracf(z)1-z=sumlimits_n=0^infty a_n z^n$.







share|cite|improve this answer














share|cite|improve this answer



share|cite|improve this answer








edited Feb 19 at 2:06

























answered Feb 18 at 16:31









SongSong

17.5k21346




17.5k21346











  • $begingroup$
    That's beautiful, thank you. I looked at the series (1-z) sum a_n z^n but didn't see how to get this contradiction.
    $endgroup$
    – Van Latimer
    Feb 18 at 16:50
















  • $begingroup$
    That's beautiful, thank you. I looked at the series (1-z) sum a_n z^n but didn't see how to get this contradiction.
    $endgroup$
    – Van Latimer
    Feb 18 at 16:50















$begingroup$
That's beautiful, thank you. I looked at the series (1-z) sum a_n z^n but didn't see how to get this contradiction.
$endgroup$
– Van Latimer
Feb 18 at 16:50




$begingroup$
That's beautiful, thank you. I looked at the series (1-z) sum a_n z^n but didn't see how to get this contradiction.
$endgroup$
– Van Latimer
Feb 18 at 16:50

















draft saved

draft discarded
















































Thanks for contributing an answer to Mathematics Stack Exchange!


  • Please be sure to answer the question. Provide details and share your research!

But avoid


  • Asking for help, clarification, or responding to other answers.

  • Making statements based on opinion; back them up with references or personal experience.

Use MathJax to format equations. MathJax reference.


To learn more, see our tips on writing great answers.




draft saved


draft discarded














StackExchange.ready(
function ()
StackExchange.openid.initPostLogin('.new-post-login', 'https%3a%2f%2fmath.stackexchange.com%2fquestions%2f3117764%2fa-power-series-with-decreasing-positive-coefficients-has-no-zeroes-in-the-disk%23new-answer', 'question_page');

);

Post as a guest















Required, but never shown





















































Required, but never shown














Required, but never shown












Required, but never shown







Required, but never shown

































Required, but never shown














Required, but never shown












Required, but never shown







Required, but never shown






Popular posts from this blog

How to check contact read email or not when send email to Individual?

Bahrain

Postfix configuration issue with fips on centos 7; mailgun relay